Clinical Cases Lippincott Pharmacology PDF
Document Details
Uploaded by Deleted User
Hiba Alzoubi
Tags
Related
Summary
This document is a collection of pharmacology clinical cases, covering topics such as pharmacokinetics, autonomic nervous system drugs, and cholinergic agonists/antagonists. The cases involve patients with various conditions and present multiple-choice questions related to drug administration, mechanisms of action, and potential side effects. The document appears to be a study guide or textbook.
Full Transcript
Clinical Cases File Lippincott Pharmacology Textbook # LippincottChallenge Total 175 Ceases Collected and Summarized By: Hiba Alzoubi...
Clinical Cases File Lippincott Pharmacology Textbook # LippincottChallenge Total 175 Ceases Collected and Summarized By: Hiba Alzoubi 1|Page Hiba Alzoubi UNIT I: Principles of Drug Therapy Chapter 1: Pharmacokinetics 1- An 18-year-old female patient is brought to the emergency department due to drug overdose. Which of the following routes of administration is the most desirable for administering the antidote for the drug overdose? A. Intramuscular. B. Subcutaneous. C. Transdermal. D. Oral. E. Intravenous. o Correct answer = E. The intravenous route of administration is the most desirable because it results in achievement of therapeutic plasma levels of the antidote rapidly. 2- A 40-year-old male patient (70 kg) was recently diagnosed with infection involving methicillin-resistant S. aureus. He received 2000 mg of vancomycin as an IV loading dose. The peak plasma concentration of vancomycin was reported to be 28.5 mg/L. The apparent volume of distribution is: A. 1 L/kg. O B. 10 L/kg. Too 228 5mg h f C. 7 L/kg. L ids 209,574 D. 70 L/kg. E. 14 L/kg. dog Correct answer = A. Vd = dose/C = 2000 mg/28.5 mg/L = 70.1 L. Because the off patient is 70 kg, the apparent volume of distribution in L/kg will be approximately 1 L/kg (70.1 L/70 kg). 3- A 65-year-old female patient (60 kg) with a history of ischemic stroke was prescribed clopidogrel for stroke prevention. She was hospitalized again after 6 months due to recurrent ischemic stroke. Which of the following is a likely reason she did not respond to clopidogrel therapy? She is a: A. Poor CYP2D6 metabolizer. V B. Fast CYP1A2 metabolizer. C. Poor CYP2E1 metabolizer. D. Fast CYP3A4Emetabolizer. E. Poor CYP2C19 metabolizer. 2|Page Hiba Alzoubi Correct answer = E. Clopidogrel is a prodrug, and it is activated by CYP2C19, which is a cytochrome P450 (CYP450) enzyme. Thus, patients who are poor CYP2C19 metabolizers have a higher incidence of cardiovascular events (for example, stroke or myocardial infarction) when taking clopidogrel. 4- A 55-year-old male patient (70 kg) is going to be treated with an experimental drug, Drug X, for an irregular heart rhythm. If the Vd is 1 L/kg and the desired steadystate plasma concentration is 2.5 mg/L, which of the following is the most appropriate intravenous loading dose for Drug X? A. 175 mg. e B. 70 mg. C. 28 mg. D. 10 mg. IDs Jd con E. 1 mg. 70 X2 55175 Correct answer = A. For IV infusion, Loading dose = (Vd) × (desired steady-state plasma concentration). The Vd in this case corrected to the patient’s weight is 70 L. Thus, Loading dose = 70 L × 2.5 mg/L = 175 mg. UNIT II: Drugs Affecting the Autonomic Nervous System Chapter 3: The Autonomic Nervous System 5- An elderly man was brought to the emergency room after he ingested a large quantity of carvedilol tablets, a drug that blocks α1, β1, and β2 adrenergic receptors, which mainly mediate the cardiovascular effects of epinephrine and norepinephrine in the body. Which of the following symptoms would you expect in To this patient? A. Increased heart rate (tachycardia). B. Reduced heart rate (bradycardia). C. Dilation of the pupil (mydriasis). D. Increased blood pressure. at B Correct answer = B. Activation of α1 receptors causes mydriasis, vasoconstriction, and an increase in blood pressure. Activation of β1 receptors increases heart rate, G of β2 receptors causes contractility of the heart, and blood pressure. Activation dilation of bronchioles and relaxation of skeletal muscle vessels. Thus, inhibition of these receptors will cause vasorelaxation (α1 blockade), reduction in heart rate (β1 blockade), reduction in contractility of the heart (β1 blockade), reduction in Con 3|Page Hiba Alzoubi blood pressure, bronchoconstriction (β2 blockade), and constriction of blood vessels supplying skeletal muscles (β2 blockade). Chapter 4: Cholinergic Agonists 6- An elderly female who lives in a farm house was brought to the emergency room in serious condition after ingesting a liquid from an unlabeled bottle found near her bed, apparently in a suicide attempt. She presented with diarrhea, frequent urination, convulsions, breathing difficulties, constricted pupils (miosis), and excessive salivation. Which of the following is correct regarding this patient? A. She most likely consumed an organophosphate pesticide. B. The symptoms are consistent with sympathetic activation. C. Her symptoms can be treated using an anticholinesterase agent. D. Her symptoms can be treated using a cholinergic agonist. Correct answer = A. The symptoms are consistent with that of cholinergic crisis. Since the elderly female lives on a farm and since the symptoms are consistent with that of cholinergic crisis (usually caused by cholinesterase inhibitors), it may be assumed that she has consumed an organophosphate pesticide (irreversible cholinesterase inhibitor). Assuming that the symptoms are caused by organophosphate poisoning, administering an anticholinesterase agent or a cholinergic agonist will worsen the condition. The symptoms are not consistent with that of sympathetic activation, as sympathetic activation will cause symptoms opposite to that of cholinergic crisis seen in this patient. Chapter 5: Cholinergic Antagonists 7- During an ophthalmic surgical procedure, the surgeon wanted to constrict the pupil of the patient using a miotic drug. However, he accidentally used another following drugs did he use? I drug that caused dilation of the pupil (mydriasis) instead. Most likely, which of the of A. Acetylcholine. I B. Pilocarpine. C. Tropicamide. a D. Phentolamine. E. Bethanechol. Correct answer = C. Muscarinic agonists such as ACh, pilocarpine, and bethanechol contract the circular muscles of iris sphincter and cause constriction of the pupil (miosis), whereas muscarinic antagonists such as atropine and ropicamide 4|Page Hiba Alzoubi prevent the contraction of the circular muscles of the iris and cause dilation of the pupil (mydriasis). α-Adrenergic antagonists such as phentolamine relax the radial muscles of the iris and cause miosis. 8- A patient with chronic obstructive pulmonary disease (COPD) was prescribed a β2 agonist for the relief of bronchospasm. However, the patient did not respond to this treatment. Which of the following drugs or classes of drugs would you suggest for this patient as the next option? A. β1 Agonist. B. Muscarinic agonist. C. Physostigmine. ooo D. Ipratropium. E. Phentolamine. Correct answer = D. Major receptors present in the bronchial tissues are muscarinic and adrenergic-β2 receptors. Muscarinic activation causes bronchoconstriction, and β2 receptor activation causes bronchodilation. Therefore, direct or indirect (physostigmine) muscarinic agonists will worsen bronchospasm. Ipratropium is a muscarinic antagonist that can relax bronchial smooth muscles and relieve bronchospasm in patients who are not responsive to β2 agonists. α1 and β1 g receptors are not commonly present in bronchial tissues and, therefore, β1 agonists or α antagonists (phentolamine) do not have any significant effects on bronchospasm. 9- A patient was administered a neuromuscular blocker (NMB) prior to a surgical procedure to produce skeletal muscle paralysis. This NMB drug affected small, rapidly contracting muscles of the face and eyes first and diaphragm muscles last. The effect of this drug was easily reversed with neostigmine. Which of the following neuromuscular blockers was most___ likely administered to this patient? _t A. Rocuronium. me B. Succinylcholine. C. Diazepam. D. Tubocurarine. Correct answer = A. There are two types of NMBs: depolarizing an nondepolarizing NMBs. Depolarizing NMBs are agonists at the nicotinic receptors, whereas nondepolarizing NMBs are antagonists at the nicotinic receptors. Both types of NMBs affect the rapidly contracting muscles (face, eye, etc.) first and diaphragm muscles last. However, cholinesterase inhibitors such as neostigmine increase ACh levels in the NMJ and reverse the effects of nondepolarizing NMBs, but not those of depolarizing NMBs. Therefore, the NMB administered to this 5|Page Hiba Alzoubi patient is most probably rocuronium, which is a nondepolarizing NMB. Tubocurarine is also a nondepolarizing NMB, but it is not used in practice. Succinylcholine is a depolarizing NMB, and diazepam is a benzodiazepine that does not cause paralysis of skeletal muscles. 10- A patient was administered a neuromuscular blocker (NMB) prior to a surgical procedure to produce skeletal muscle paralysis. This NMB drug caused initial skeletal muscle fasciculations before the onset of paralysis. The effect of this drug could not be reversed with neostigmine. Which of the following neuromuscular blockers was most likely administered to this patient? A. Cisatracurium. tsar B. Succinylcholine. C. Diazepam. D. Tubocurarine. Correct answer = B. Depolarizing NMBs cause muscle fasciculations before causing paralysis, and their effects cannot be reversed using cholinesterase inhibitors such as neostigmine. Nondepolarizing NMBs do not cause muscle fasciculations, and their effects can be reversed using cholinesterase inhibitors. Therefore, the NMB used in this patient is succinylcholine, which is a depolarizing NMB. Cisatracurium and tubocurarine are nondepolarizing NMBs, and diazepam does not cause paralysis of skeletal muscles. Chapter 6: Adrenergic Agonists 11- A hypertensive patient was accidentally given an α2 agonist instead of an α1 blocker. Which of the following is correct in this situation? A. α2 Agonists can increase the release of norepinephrine from sympathetic nerve terminals. B. α2 Agonists can reduce blood pressure in this patient. C. α2 Agonists can increase blood pressure in this patient. D. α2 Agonists will not affect blood pressure in this patient. Correct answer = B. α2 Agonists activate α2 receptors located in the presynaptic terminal of sympathetic neurons and cause a reduction in the release of norepinephrine from sympathetic nerve terminals. This leads to a reduction in blood pressure. α2 Agonists such as clonidine and methyldopa are therefore used as antihypertensive agents. 6|Page Hiba Alzoubi Brats p Tay H TWIN f 12- An asthma patient was given a nonselective β agonist to relieve Ii bronchoconstriction. Which of the following adverse effects would you expect to see in this patient? A. Bradycardia. B B. Tachycardia. AT C. Hypotension (reduction in blood pressure). a on Bloc D. Worsening bronchoconstriction. Correct answer = B. A nonselective β agonist activates both β1 as well as β2 __ receptors. β1 activation causes an increase __ in heart rate (tachycardia), contractility, and subsequent increase in blood pressure. It relieves bronchoconstriction because of the β2 receptor activation. 13- A 12-year-old boy who is allergic to peanuts was brought to the emergency room after accidentally consuming peanuts contained in fast food. He is in anaphylactic shock. Which of the following drugs would be most appropriate to treat this patient? A. Norepinephrine. B. Phenylephrine. C. Dobutamine. OO D. Epinephrine. Correct answer = D. Norepinephrine has more α agonistic effects and activates mainly α1, α2, and β1 receptors. Epinephrine has more β agonistic effects and activates mainly α1, α2, β1, and β2 receptors. Phenylephrine has predominantly α effects and activates mainly α1 receptors. Dobutamine mainly activates β1 receptors and has no significant effects on β2 receptors. Thus, epinephrine is the drug of choice in anaphylactic shock that can both stimulate the heart (β1 activation) and dilate bronchioles (β2 activation). 14- A 70-year-old patient was brought to the emergency room with a blood pressure of 76/60 mm Hg, tachycardia, and low cardiac output. He was diagnosed with acute heart failure. Which of the following drugs would be the most appropriate to improve his cardiac function? A. Epinephrine. B. Fenoldopam. C. Dobutamine. con a E D. Isoproterenol. l a Hiba Alzoubi net 7|Page ox Correct answer = C. Among the choices, the ideal drug to increase contractility of the heart in acute heart failure is dobutamine, since it is a selective β1-adrenergic agonist. Fenoldopam is a dopamine agonist used to treat severe hypertension. Other drugs are nonselective adrenergic agonists that could cause unwanted side effects. 15- One of your patients who is hypertensive and gets mild asthma attacks 5 occasionally bought an herbal remedy online to help with his asthma. He is not on any asthma medications currently but is receiving a β1-selective blocker for his hypertension. The herbal remedy seems to relieve his asthma attacks, but his blood pressure seems to increase despite the β-blocker therapy. Which of the following drugs is most likely present in the herbal remedy he is taking? A. Phenylephrine. B. Norepinephrine. b C. Dobutamine. D. Ephedrine. ay Pat Bagonist E. Salmeterol. as Correct answer = D. Two drugs among the choices that could relieve asthma are ephedrine and salmeterol, as they activate β2 receptors in the bronchioles and cause bronchodilation. However, salmeterol is a selective β2 agonist and should not cause an increase in blood pressure. Ephedrine on the other hand stimulates the release of norepinephrine and acts as a direct agonist at α- and β-adrenergic receptors, thus causing an increase in blood pressure. Phenylephrine (a nonselective α agonist) does not cause bronchodilation. Norepinephrine is a nonselective adrenergic agonist that does not have any stimulatory effects on β2 receptors. Also, norepinephrine is not active when given orally. Chapter 7: Adrenergic Antagonists 16- A 30-year-old male patient was brought to the ER with amphetamine overdose. He presented with high blood pressure and arrhythmia. Which of the following is correct regarding this patient? opinion A. Amphetamine can activate all types of adrenergic receptors. B. β-Blockers are the ideal antidotes for amphetamine poisoning. C. α-Blockers can normalize the blood pressure in this patient. agonist D. Miosis could be a possible symptom of amphetamine poisoning. for ist Correct answer = A. Amphetamine is an indirect adrenergic agonist that mainly enhances the release of norepinephrine from peripheral sympathetic neurons. 8|Page Hiba Alzoubi Therefore, it activates all types of adrenergic receptors (that is, α and β receptors) and causes an increase in blood pressure. Since both α and β receptors are activated by amphetamine, α-blockers or β-blockers alone cannot relieve the symptoms of amphetamine poisoning. Since amphetamine causes sympathetic activation, it causes mydriasis, not miosis. 17- A β-blocker was prescribed for hypertension in a female asthma patient. After about a week of treatment, the asthma attacks got worse, and the patient was asked to stop taking the β-blocker. Which of the following β-blockers would you suggest as an alternative in this patient that is less likely to worsen her asthma? A. Propranolol. B. Metoprolol. non T C. Labetalol. D. Carvedilol. Correct answer = B. The patient was most likely given a nonselective β-blocker (antagonizes both β1 and β2 receptors) that made her asthma worse due to β2 antagonism. An alternative is to prescribe a cardioselective (antagonizes only β1) β-blocker that does not antagonize β2 receptors in the bronchioles. Metoprolol is a cardioselective β-blocker. Propranolol, labetalol, and carvedilol are nonselective β- blockers and could worsen the asthma. 18- A 70-year-old male needs to be treated with an α-blocker for overflow incontinence due to his enlarged prostate. Which of the following drugs would you suggest in this patent that will not affect his blood pressure significantly? A. Prazosin. Phx Potensio B. Doxazosin. C. Phentolamine. Essen D. Tamsulosin. s E. Terazosin. Correct answer = D. Tamsulosin is an α1 antagonist that is more selective to the α1 receptor subtype (α1A) present in the prostate and less selective to the α1 receptor subtype (α1B) present in the blood vessels. Therefore, tamsulosin does not affect blood pressure significantly. Prazosin, doxazosin, terazosin, and phentolamine antagonize both these subtypes and cause significant hypotension as a side effect. 9|Page Hiba Alzoubi 19- A 50-year-old male was brought to the emergency room after being stung by a hornet. The patient was found to be in anaphylactic shock, and the medical team tried to reverse the bronchoconstriction and hypotension using epinephrine. However, the patient did not fully respond to the epinephrine treatment. The g patient’s wife mentioned that he is taking a prescription medication for his blood pressure, the name of which she does not remember. Which of the following medications is he most likely taking that could have prevented the effects of epinephrine? 05 at A. Doxazosin. B. Propranolol. C. Metoprolol. Bri Correct answer = B.Broncho D. Acebutolol. 14 Epinephrine reverses hypotension by activating β1 receptors B B and relieves bronchoconstriction by activating β2 receptors in anaphylaxis. Since epinephrine was not effective in reversing hypotension or bronchoconstriction in this patient, it could be assumed that the patient was on a nonselective β-blocker (propranolol). Doxazosin (α1-blocker), metoprolol, or acebutolol (both β1- selective blockers) would not have completely prevented the effects of epinephrine. Unit III: Drugs Affecting the Central Nervous System Chapter 8: Drugs for Neurodegenerative Diseases 20- Peripheral adverse effects of levodopa, including nausea, hypotension, and cardiac arrhythmias, can be diminished by including which of the following drugs in the therapy? A. Amantadine. B. Ropinirole. C. Carbidopa. 0 D. Tolcapone. 661 Way E. Pramipexole. e in yea Correct answer = C. Carbidopa inhibits the peripheral decarboxylation of levodopa to dopamine, thereby diminishing the gastrointestinal and cardiovascular side I effects of levodopa. The other agents listed do not ameliorate adverse effects of levodopa. 10 | P a g e Hiba Alzoubi Chapter 9: Anxiolytic and Hypnotic Drugs 21- A 45-year-old man who has been injured in a car accident is brought into the emergency room. His blood alcohol level on admission is 275 mg/dL. Hospital records show a prior hospitalization for alcohol-related seizures. His wife confirms that he has been drinking heavily for 3 weeks. What treatment should be provided to the patient if he goes into withdrawal? A. None. B. Lorazepam. C. Pentobarbital. Tezari IB D. Phenytoin. E. Buspirone. a 3 Correct answer = B. It is important to treat the seizures associated with alcohol withdrawal. Benzodiazepines, such as chlordiazepoxide, diazepam, or the shorter- acting lorazepam, are effective in controlling this problem. They are less sedating than pentobarbital or phenytoin. 22- Which agent is best used in the Emergency Room setting for patients who are believed to have received too much of a benzodiazepine drug or taken an overdose of benzodiazepines? A. Diazepam. B. Ramelteon. C. Flumazenil. D. Doxepin. E. Naloxone. Correct answer = C. Flumazenil is only indicated to reverse the effects of benzodiazepines via antagonizing the benzodiazepine receptor. It should be used with caution due to a risk of seizures if the patient has been a long time recipient of benzodiazepines or if the overdose attempt was with mixed drugs. Naloxone is an opioid receptor antagonist. The other agents are not efficacious in reversing effects of benzodiazepines. i Chapter 10: Antidepressants 23- A 55-year-old teacher began to experience changes in mood. He was losing interest in his work and lacked the desire to play his daily tennis match. He was preoccupied with feelings of guilt, worthlessness, and hopelessness. In addition to the psychiatric symptoms, the patient complained of muscle aches throughout his body. Physical and laboratory tests were unremarkable. After 6 weeks of therapy 11 | P a g e Hiba Alzoubi with fluoxetine, his symptoms resolved. However, the patient complains of sexual It Which of the following drugs might be useful in this patient? dysfunction. A. Fluvoxamine. s B. Sertraline. É C. Citalopram. D. Mirtazapine.s a 92 Ftw E. Lithium. as ten If Correct answer = D. Mirtazapine is largely free from sexual side effects. However, sexual dysfunction commonly occurs with SSRIs (fluvoxamine, sertraline, and citalopram), as well as with TCAs, and SNRIs. Lithium is used for the treatment of mania and bipolar disorder. 24- A 25-year-old woman has a long history of depressive symptoms accompanied by body aches and pain secondary to a car accident 2 years earlier. Physical and o laboratory tests are unremarkable. Which of the following drugs might be useful in this patient? 3465 A. Fluoxetine. B. Sertraline. Is 68 C. Phenelzine. D. Mirtazapine. no E. Duloxetine. E Correct answer = E. Duloxetine is a SNRI that can be used for depression accompanied by symptoms of pain. SSRIs (fluoxetine and sertraline), MAOIs (phenelzine), and atypical antidepressants (mirtazapine) have little activity against pain syndromes. 25- A 51-year-old woman with symptoms of major depression also has angle- closure glaucoma. Which of the following antidepressants should be avoided in this patient? I its b A. Amitriptyline. B. Sertraline. C. Bupropion. www D. Mirtazapine. E. Fluvoxamine. Close Correct answer = A. Because of its potent antimuscarinic activity, amitriptyline should not be given to patients with glaucoma because of the risk of acute to 12 | P a g e Hiba Alzoubi increases in intraocular pressure. The other antidepressants all lack antagonist activity at the muscarinic receptor. to 26- A 36-year-old man presents with symptoms of compulsive behavior. If anything is out of order, he feels that “work will not be accomplished effectively or efficiently.” He realizes that his behavior is interfering with his ability to accomplish his daily tasks but cannot seem to stop himself. Which of the following drugs would be most helpful to this patient? A. Imipramine. Tis B. Fluvoxamine. C. Amitriptyline. D. Tranylcypromine. Core E. Lithium. Correct answer = B. SSRIs are particularly effective in treating obsessive compulsive disorder, and fluvoxamine is approved for this condition. The other drugs are less effective in the treatment of obsessive–compulsive disorder. o t 27- Which agent would be a poor choice in a 70-yearold elderly female with depressive symptoms due to the drug having significant α1 receptor antagonism E and thus a higher risk for falls due to orthostatic hypotension? A. Lithium. B. Bupropion. C. Escitalopram. D. Imipramine. D E. Sertraline. at MY Correct answer = D. Lithium should not be used for depression in an elderly patient without first trying first-line antidepressants, and even then, it is used as an adjunct. Bupropion, sertraline,t and escitalopram have very little effect on blood pressure (no α1 receptor antagonism) and are considered acceptable choices for the treatment of depression in the elderly. Imipramine is associated with a high risk for orthostasis in the elderly and should be avoided due to its adverse effect profile and risk for falls. Chapter 11: Antipsychotic Drugs 28- A 21-year-old male has recently begun pimozide therapy for Tourette disorder. His parents bring him to the emergency department. They describe that he has been 13 | P a g e Hiba Alzoubi Bete having “different-appearing tics” than before, such as prolonged contraction of the facial muscles. While being examined, he experiences opisthotonos (type of extrapyramidal spasm of the body in which the head and heels are bent backward and the body is bowed forward). Which of the following drugs would be beneficial to in reducing these symptoms? A. Benztropine. B. Bromocriptine. C. Lithium. D. Prochlorperazine. IsentXDopamin E. Risperidone. Correct answer = A. The patient is experiencing EPS due to pimozide, and a muscarinic antagonist such as benztropine would be effective in reducing the symptoms. The other drugs would have no effect or, in the case of prochlorperazine and risperidone, might increase the symptoms. a 29- A 28-year-old woman with schizoaffective disorder (combination of mood and psychotic symptoms) reports difficulty falling asleep. Which of the following would be most beneficial in this patient? A. Lithium. B. Chlorpromazine. I C. Haloperidol. 5909031134 D. Paliperidone. E. Ziprasidone. _Secor insomnia Correct answer = D. Paliperidone is the only agent that is FDA approved for schizoaffective disorder. Chlorpromazine has significant sedative activity as well as antipsychotic properties and is the drug most likely to alleviate this patient’s major complaint of insomnia. Although other antipsychotics may benefit this patient’s disorder, paliperidone has the indication for this disorder, and if the underlying disorder is improved, then the symptom of insomnia may also I improve without risking other, unwanted adverse effects, such as the anticholinergic effects of chlorpromazine. 30- A 30-year-old male patient who is treated with haloperidol for his diagnosis of schizophrenia is considered to be well-managed symptomatically for his psychotic symptoms. However, he is reporting restlessness, the inability to sit still at the dinner table, and his family notices that he is pacing up and down the hallway frequently. Of the following, which is the best medication to treat this antipsychotic-induced akathisia? propan is of 14 | P a g e Hiba Alzoubi pesade C 94 Epo pad A. Benztropine. B. Dantrolene. Pinot C. Amoxapine. D. Bromocriptine. muscle milgnant E. Propranolol. Correct answer = E. Propranolol, a β-blocker, is considered the drug of choice for the management of antipsychotic- induced akathisia. Benztropine is more effective for pseudoparkinsonism and acute dystonias. Amoxapine is an antidepressant that has been associated with EPS. Bromocriptine is more effective for Parkinson-like symptoms, and dantrolene is a muscle relaxant that is best reserved for managing some symptoms of neuroleptic malignant syndrome. Chapter 12: Drugs for Epilepsy 31- A 9-year-old boy is sent for neurologic evaluation because of episodes of apparent inattention. Over the past year, the child has experienced episodes during which he develops a blank look on his face and his eyes blink for 15 seconds. He immediately resumes his previous activity. Which one the following best describes this patient’s seizures? Tess A. Simple partial. B. Complex partial. C. Tonic–clonic. D. Absence. E. Myoclonic. Correct answer = D. The patient is experiencing episodes of absence seizures. Consciousness is impaired briefly and they generally begin in children aged 4 to 12 years. Diagnosis includes obtaining an EEG that shows generalized 3-Hz waves. two 32- A child is experiencing absence seizures that interrupt his ability to pay attention during school and activities. Which of the following therapies would be most appropriate for this patient? A. Ethosuximide. B. Carbamazepine. 7 nots C. Diazepam. D. Carbamazepine plus primidone. E. Watchful waiting. 0 0 n Correct answer = A. The patient has had many seizures that interrupt his ability to absentpay attention during school and activities, so therapy is justified. Monotherapy 15 | P a g e Hiba Alzoubi with primary agents is preferred for most patients. The advantages of monotherapy include reduced frequency of adverse effects, fewer interactions between antiepileptic drugs, lower cost, and improved compliance. Carbamazepine and diazepam are not indicated for absence seizures. 33- A 25-year-old woman with myoclonic seizures is well controlled on valproate. She indicates that she is interested in becoming pregnant in the next year. With respect to her antiepilepsy medication, which of the following should be considered? A. Leave her on her current therapy. B. Consider switching to lamotrigine. C. Consider adding a second antiepilepsy medication. D. Decrease her valproate dose. x Correct answer = B. Valproate is a poor choice in women of child-bearing age. A review of the medication history of this patient is warranted. If she has not tried any other antiepilepsy medication, then consideration of another antiepilepsy medication may be beneficial. Studies show that valproate taken during pregnancy can have a detrimental effect on cognitive abilities in children. 34- A woman with myoclonic seizures is well controlled with lamotrigine. She becomes pregnant and begins to have breakthrough seizures. What is most likely happening? A. Her epilepsy is getting worse. B. Lamotrigine concentrations are increasing. At C. Lamotrigine concentrations are decreasing. D. Lamotrigine is no longer efficacious for this patient. Correct answer = C. Pregnancy alters the pharmacokinetics of lamotrigine. As pregnancy progresses, most women require increased dosages to maintain blood concentrations and seizure control. 50 35- A 42-year-old man undergoes a neurologic evaluation because of episodes of apparent confusion. Over the past year, the man has experienced episodes during which he develops a blank look on his face and fails to respond to questions. Moreover, it appears to take several minutes before the man recovers from the episodes. Which one of the following best describes this type of seizure? A. Focal (simple partial). B. Focal (complex partial). Hiba Alzoubi In C. Tonic–clonic. 16 | P a g e D. Absence. E. Myoclonic. Correct answer = B. The patient is experiencing episodes of complex partial seizures. Complex partial seizures impair consciousness and can occur in all age groups. Typically, staring is accompanied by impaired consciousness and recall. If asked a question, the patient might respond with an inappropriate or unintelligible answer. Automatic movements are associated with most complex partial seizures and involve the mouth and face (lip-smacking, chewing, tasting, and swallowing movements), upper extremities (fumbling, picking, tapping, or clasping movements), vocal apparatus (grunts or repetition of words and phrases), as are complex acts (such as walking or mixing foods in a bowl). 36- A 52-year-old man has had several focal complex partial seizures over the last year. Which one of the following therapies would be the most appropriate initial therapy for this patient? A. Ethosuximide. B. Levetiracetam. C. Diazepam. D. Carbamazepine plus primidone. E. Watchful waiting. a atonal 4.4 Correct answer = B. The patient has had many seizures, and the risks of not I starting drug therapy would be substantially greater than the risks of treating his seizures. Because the patient has impaired consciousness during the seizure, he is at risk for injury during an attack. Monotherapy with primary agents is preferred for most patients. The advantages of monotherapy include reduced frequency of adverse effects, absence of interactions between antiepileptic drugs, lower cost, and improved compliance. Ethosuximide and diazepam are not indicated for complex partial seizures. I 37- A patient with focal complex partial seizures has been treated for 6 months with carbamazepine but, recently, has been experiencing breakthrough seizures on a more frequent basis. You are considering adding a second drug to the antiseizure regimen. Which of the following drugs is least likely to have a pharmacokinetic interaction with carbamazepine? A. Topiramate. B. Tiagabine. C. Levetiracetam. 17 | P a g e Hiba Alzoubi D. Lamotrigine. E. Zonisamide. t I Correct answer = C. Of the drugs listed, all of which are approved as adjunct therapy for refractory focal complex partial seizures, only levetiracetam does not affect the pharmacokinetics of other antiepileptic drugs, and other drugs do not significantly alter its pharmacokinetics. However, any of the listed drugs could be added depending on the plan and the patient characteristics. Treatment of epilepsy is complex, and diagnosis is based on history and may need to be reevaluated when drug therapy fails or seizures increase. 38- A 75-year-old woman had a stroke approximately 1 month ago. She is continuing to have small focal seizures where she fails to respond appropriately while talking. Which of the following is the most appropriate treatment for this individual? A. Phenytoin. B. Oxcarbazepine. C. Levetiracetam. T M'm bra bratration any D. Phenobarbital. integerCorrect answer = C. Levetiracetam is renally cleared and prone to very few drug interactions. Elderly patients usually have more comorbidities and are taking more in eidt medications than younger patients. Oxcarbazepine may cause hyponatremia, which is more symptomatic in the elderly.Phenytoin and phenobarbital have many drug interactions and a side effect profile that may be especially troublesome in the elderly age group including dizziness that may lead to falls, cognitive issues, and bone health issues. Chapter 13: Anesthetics 39- A patient with heart failure and significantly reduced cardiac output requires surgical anesthesia. Which of the following would you expect to see in this patient? A. Slower induction time with IV anesthetics. O B. Need for increased dosage of IV anesthetics. C. Slower induction time with inhaled anesthetics. D. Enhanced removal of inhaled anesthetics to peripheral tissues. 18 | P a g e Hiba Alzoubi Correct answer = A. When cardiac output is reduced, the body compensates by diverting more cardiac output to the cerebral circulation. A greater proportion of the IV anesthetic enters the cerebral circulation under these circumstances. Therefore, the dose of the IV drug must be reduced (not increased). Also, with reduced cardiac output, it takes a longer time for an IV induction drug to reach the brain, resulting in a slower induction time. For inhaled anesthetics, lower cardiac output removes anesthetic from the alveoli to the peripheral tissues more slowly and thus enhances the rate of rise in alveolar concentration of gas. Therefore, the gas reaches equilibrium between the alveoli and the site of action in the brain more quickly. 40- An 80-year-old patient with asthma and low blood pressure requires anesthesia for an emergency surgical procedure. Which of the following agents would be bait most appropriate for inducing anesthesia in this patient? A. Desflurane. j B. Ketamine. C. Propofol. Asthma THIN fat D. Thiopental. barbital 5.3 1 Correct answer = B. Ketamine may be beneficial since it is a potent bronchodilator and may not lower blood pressure like other agents. Desflurane is an inhaled anesthetic that may stimulate respiratory reflexes. It is used for maintenance, not induction, and may lower blood pressure. Propofol may also decrease blood T pressure. Thiopental is a short-acting barbiturate that can cause bronchospasm. 41- A 52-year-old woman will be undergoing sedation with propofol for a brief diagnostic procedure. Which of the following is an advantage of propofol for this patient? A. Rapid analgesia. B. Sustained duration. C. Decreased incidence of nausea and vomiting. is D. Less pain at the injection site. Correct answer = C. Propofol has some antiemetic effect, so it does not cause postoperative nausea and vomiting. I has a short duration of action (which makes it good for brief procedures), but does not produce analgesia. Pain at the injection site is common. 42- A 32-year-old woman requests an epidural to ease labor pains. She reports that she had an allergic reaction to Novocain (procaine) at the dentist’s office. Which of 19 | P a g e Hiba Alzoubi the following local anesthetics would be appropriate for use in an epidural for this patient? A. Chloroprocaine. B. Mepivacaine. mid toxic in non at proof C. Ropivacaine. D. Tetracaine. II Correct answer = C. Procaine is an ester local anesthetic. Since this patient has an allergy to procaine, other ester anesthetics (chloroprocaine, tetracaine) should not be used. Mepivacaine, an amide local anesthetic, should not be used due to the potential for increased toxicity to the neonate. Ropivacaine is an amide anesthetic. I __ Chapter 14: Opioids 43- A young woman is brought into the emergency room. She is unconscious, and she has pupillary constriction and depressed respiration. Based on reports, an opioid overdose is almost certain. Which of the listed phenanthrene opioids will exhibit a full and immediate response to treatment with naloxone? A. Meperidine. B. Morphine. C. Buprenorphine. D. Fentanyl. nolo to a nabroxin 8 a Correct answer = B. A morphine overdose can be effectively treated with naloxone, and morphine is a phenanthrene. Naloxone antagonizes the opioid by displacing it from the receptor, but there are cases in which naloxone is not effective. Meperidine is a phenylpiperidine, not a phenanthrene, and the active metabolite, normeperidine, is not reversible by naloxone. The effects of uprenorphine are only partially reversible by naloxone. Naloxone is effective for fentanyl overdoses; however, fentanyl is a phenylpiperidine, and not a phenanthrene. 44- A 76-year-old female with renal insufficiency presents to the clinic with severe pain secondary to a compression fracture in the lumbar spine. She reports that the pain has been uncontrolled with tramadol, and it is decided to start treatment with an opioid. Which of the following is the best opioid for this patient? A. Meperidine. C. Hydrocodone. D. Morphine. Vig B. Fentanyl transdermal patch. win 20 | P a g e Hiba Alzoubi Correct answer = C. Hydrocodone would be the best choice of the opioid given in this case. It will be very important to use a low dose and monitor closely for proper pain control and any side effects. Meperidine should not be used for chronic pain, nor should it be used in a patient with renal insufficiency. The transdermal patch is not a good option, since at this time, her pain would be considered acute and she is opioid naïve. Morphine also is not the best choice in this case due to the active I metabolites that can accumulate in renal insufficiency. 45- A 56-year-old patient who has suffered with severe chronic pain with radiculopathy secondary to spinal stenosis for years presents to the clinic for pain management. Over the years, this patient has failed to receive relief from the neuropathic pain from the radiculopathy with traditional agents such as tricyclics or anticonvulsants. Based on the mechanism of action, which opioid might be t beneficial in this patient to treat both nociceptive and neuropathic pain? A. Meperidine. I B. Oxymorphone. tain C. Morphine. nee Pathe D. Methadone. aphid andpain Correct answer = D. Methadone has a unique mechanism of action in comparison to the other choices given. Methadone is a μ agonist, but it also exhibits NMDA 1 receptor antagonism that is thought to aid in the treatment of neuropathic pain and I a could also aid in prevention of opioid tolerance. All other μ agonists could help 11 manage neuropathic pain, but in some situations, higher doses of opioids are needed to achieve efficacy. It is much better to consider adjuvants such as tricyclics or certain anticonvulsants in the treatment of neuropathic pain. 46- A 64-year-old male is preparing for a total knee replacement. He is taking many medications that are metabolized by the CYP450 enzyme system and is morph e worried about drug interactions with the pain medication that will be used following his surgery. Which of the following opioids would have the lowest chance of interacting with his medications that are metabolized by the CYP450 enzyme system? or A. Methadone. B. Oxymorphone. t C. Oxycodone. D. Hydrocodone. odd cap 450 Correct answer = B. Oxymorphone is metabolized via glucuronidation and has not been shown to have any drug interactions associated with the CYP enzyme family. 21 | P a g e Hiba Alzoubi All other opioids listed are metabolized by one or more CYP enzymes and increase the risk of drug interactions. 47- KM is a 64-year-old male who has been hospitalized following a car accident in which he sustained a broken leg and broken arm. He has been converted to oral morphine in anticipation of his discharge. What other medication should he receive with his morphine upon discharge? A. Diphenhydramine. urticaria a gym B. Methylphenidate. sedateC. Docusate sodium with Senna. 44 D. Docusate sodium. 08 as Correct answer = C. A bowel regimen should be prescribed with the initiation of the opioid. Docusate and senna include both a stool softener and a stimulant, which is recommended for opioid-induced constipation. Treatment with docusate sodium only is ineffective. Constipation is very common with all opioids, a and tolerance does not occur. Diphenhydramine can be used for urticaria that might occur with the initiation of an opioid, but this is not reported in this case. Methylphenidate has been used for opioid-induced sedation in certain situations, but is not an issue in this case. t 48- AN is a 57-year-old male who has been treated with oxycodone for chronic Iet nonmalignant pain for over 2 years. He is now reporting increased pain in the t work. Which of the following opioids is a short-acting opioid afternoon while at and is the best choice for this patient’s breakthrough pain? A. Methadone. B. Pentazocine. Ia C. Hydrocodone. D. Nalbuphine. Correct answer = C. Hydrocodone is a commonly used short-acting agent that is commercially available in combination form with either acetaminophen or ibuprofen. Methadone should not routinely be used for breakthrough pain due to the unique pharmacokinetics and should be reserved for practitioners who have experience with this agent and understand the variables associated with this drug. Pentazocine and nalbuphine are mixed agonist/antagonist analgesics that could precipitate withdrawal in patients who are currently taking a tu full μ agonist such as oxycodone. 22 | P a g e Hiba Alzoubi Chapter 15: Drugs of Abuse 49- A 22-year-old HIV patient has been told that marijuana may benefit him should he start using the substance. Which of the following adverse effects has been associated with marijuana usage and may be a reason for this patient to avoid use of marijuana? A. Hyperphagia. 5 B. Hyperthermia. C. Hepatitis. D. Progression of HIV. WEI if E. Hyponatremia. MR Correct answer = D. Although hyperphagia is a side effect observed with marijuana usage, this may be of benefit for some HIV patients. Hyperthermia, hepatitis, and hyponatremia have not been associated with marijuana use. Progression of HIV has been linked to marijuana use and is a serious consideration for anyone with this disease. 50- A 21-year-old college student is curious about the effects of LSD. She asks what type of risks may be involved with using the drug for the firstt time. Which of the following is a correct response to her question? A. Exaggerated hallucinations. B. Cardiomyopathy. C. Hyperphagia. LS Dad oh D. Bronchitis. Lysergic acid diethylamide Correct answer = A. Exaggerated hallucinations, sometimes known as “bad trips,” may occur, even in first-time__users. These hallucinations can lead to extreme panic, which has caused individuals to react in a manner very uncharacteristic of their typical behavior. 51- A 58-year-old male is brought into the emergency department following an automobile accident. His blood alcohol level on admission is 280 mg/dL. He has been treated in the past for seizures related to alcohol abuse, and he confirms that he has been drinking heavily over the past month since losing his job. What treatment should be given to this patient if he begins to go into withdrawal while hospitalized? A. None. B. Lorazepam. O lorazepam C. Acamprosate. Hiba Alzoubi sear andwith drool 23 | P a g e It D. Naltrexone. E. Disulfiram. Correct answer = B. Should this patient go into alcohol withdrawal, he will likely also have seizures associated with it, given his past history. Benzodiazepines are used to treat seizures associated with alcohol withdrawal. Acamprosate, naltrexone, and disulfiram may be considered at a later time to treat the dependence, but would not be useful in the acute withdrawal setting. 52- A 35-year-old man has been abusing cocaine and is agitated, tachycardic, hypertensive, and hyperthermic. Which of the following is correct regarding treatment in this situation? A. This patient should undergo gastric lavage; that is, he should have his stomach pumped immediately. B. Cocaine toxicity commonly involves CNS depression that can be reversed with IV atropine. j.se C. Benzodiazepines would be a good choice, as they should help calm the patient down, decrease heart rate, decrease blood pressure, and decrease body temperature. D. Phenobarbital should be the first choice as an anticonvulsant. Correct answer = C. Benzodiazepines such as lorazepam have anxiolytic properties and can calm a cocaine toxic patient down, thereby decreasing heart rate and blood pressure. As the patient becomes less agitated, he/she decreases movement and his or her body temperature drops. In addition, the use of benzodiazepines decreases the chance of the patient experiencing a convulsion and would be the first choice to treat cocaine-induced convulsions. 53- A 22-year-old man with a history of substance abuse arrives in the emergency department hypertensive, hyperthermic, and tachycardic, with altered mental status and hyperreflexia. His friends say he has been snorting “bath salts.” Which of the following is correct regarding this patient? A. This patient’s clinical presentation is consistent with opioid toxicity and he should receive an opioid antagonist such as naloxone immediately. to B. “Bath salts” are often labeled as “not for human consumption” and sold with an unstated understanding that they contain synthetic cathinones, which are amphetamine-like compounds. C. Treatment with a serotonin agonist might be beneficial. D. Along with cooling measures, antihypertensives, β-blockers, and monoamine oxidase inhibitors would be reasonable options for the treatment. 24 | P a g e Hiba Alzoubi Correct answer = B: “Bath salts” often contain synthetic cathinones and are labeled, marketed, and sold as something “not for human consumption” to avoid law enforcement and prosecution. In addition, they are usually not detected on urine toxicology screening. These products can cause an amphetamine-like sympathomimetic toxidrome, as well as serotonin syndrome, which would be treated with symptomatic/ supportive care and possibly a serotonin antagonist (not a serotonin agonist) such as cyproheptadine. The combination of an amphetamine or amphetamine-like substance and a monoamine oxidase inhibitor (MAO inhibitor) can precipitate serotonin syndrome and should be avoided in a hyperdynamic patient such as this. Chapter 16: CNS Stimulants 54- A young male was brought to the emergency room by the police due to severe agitation. Psychiatric examination revealed that he had injected dextroamphetamine several times in the past few days, the last time being 10 hours previously. He was given a drug that sedated him, and he fell asleep. Which of the following drugs was most likely used to counter this patient’s apparent symptoms of dextroamphetamine withdrawal? Af lol A. Phenobarbital. B. Lorazepam. Saadhargic mouse wa A C. Cocaine. D. Hydroxyzine. E. Fluoxetine. e mitis Correct answer = B. The anxiolytic properties of benzodiazepines, such as lorazepam, make them the drugs of choice in treating the anxiety and agitation of amphetamine or cocaine abuse. Lorazepam also has hypnotic properties. Phenobarbital has hypnotic properties, but its anxiolytic properties are inferior to those of the benzodiazepines. Hydroxyzine, an antihistamine, is effective as a hypnotic, and it is sometimes used to deal with anxiety, especially if emesis is a problem. Fluoxetine is an antidepressant with no immediate effects on anxiety or agitation. 55- JM is a 10-year-old male who is sent to a pediatric neurologist for an evaluation due to receiving poor grades in class. JM’s parents have recently received complaints from his teacher that he is performing poorly in school and he is repeatedly caught not paying attention in class. Several times a day during class, JM is noted to be getting out of his chair and socializing with other students. He has also been getting into fights with some children, as he is being 25 | P a g e Hiba Alzoubi singled out by others and teased. JM is given a diagnosis of ADHD with impulsivity and irritability. Which of the following is the most appropriate recommendation for management of the ADHD? A. Clonidine. B. Caffeine. C. Dextroamphetamine. D. Haloperidol. É E. Buspirone. Correct answer = C. Dextroamphetamine is the only stimulant medication in the list that is approved for ADHD. Certain symptoms like fighting may improve with haloperidol and hyperactivity may improve with clonidine, but these agents would not improve the patient’s academic performance and the underlying problems. 56- JM is a 10-year-old male with ADHD. His symptoms are currently controlled with an oral psychostimulant. However, he and his family wish to avoid having to give a second dose of medication at school. They are looking for an alternative treatment option that could be implemented in the morning and last the entire day. Which treatment option would be best for JM’s needs? A. Mixed amphetamine salts in immediate-release oral tablet formulation. t t A B. Methylphenidate in a transdermal delivery system. C. Nicotine in a chewing gum formulation for buccal absorption. D. Methylphenidate in immediate-release pills. izaw Correct answer = B. Methylphenidate is also a psychostimulant, and the C transdermal (patch) formulation is designed for once-per-day use to avoid middle of the day dosing. Immediate-release formulations require dosing at least twice daily. Nicotine is not indicated for ADHD. 57- TT is a 35-year-old male who is interested in quitting smoking. In previous quit attempts, he has tried nicotine gum, the nicotine patch, and the “cold turkey” method. He has been unsuccessful in each of these attempts and usually resumed smoking within 4 to 6 weeks. Which of the following may be useful to assist TT in his attempt to quit smoking? A. Varenicline. Erm B. Dextroamphetamine. raviningad C. Lorazepam. D. Methylphenidate. varinidne 26 | P a g e Hiba Alzoubi Correct answer = A. Varenicline is FDA approved as an adjunctive treatment option for the management of nicotine dependence. It is believed to attenuate the withdrawal symptoms of smoking cessation, though continued observation is needed to monitor for changes in psychiatric status, including suicidal ideation. The use of dextroamphetamine, lorazepam, and methylphenidate will bring the risk of addiction to another substance with abuse potential. UNIT IV: Drugs Affecting the Cardiovascular System Chapter 17: Antihypertensives 58- A 45-year-old man was just started on therapy for hypertension and developed a persistent, dry cough. Which is most likely responsible for this side effect? A. Enalapril. B. Losartan. C. Nifedipine. D. Prazosin. E. Propranolol. Correct answer = A. The cough is most likely an adverse effect of the ACE inhibitor enalapril. Losartan is an ARB that has the same beneficial effects as an ACE inhibitor but is less likely to produce a cough. Nifedipine, prazosin, and propranolol does not cause this side effect. 59- A 48-year-old hypertensive patient has been successfully treated with a thiazide diuretic for the last 5 years. Over the last 3 months, his diastolic pressure has steadily increased, and he was started on an additional antihypertensive agent. He complains of several instances of being unable to achieve an erection and not being able to complete three sets of tennis as he once did. Which is the likely second antihypertensive medication? A. Captopril. B. Losartan. C. Metoprolol. D. Minoxidil. E. Nifedipine. seal disfaction Correct answer = C. The side effect profile of β-blockers, such as metoprolol, is characterized by interference with sexual performance and decreased exercise tolerance. None of the other drugs is likely to produce this combination of side effects. 27 | P a g e Hiba Alzoubi 60- A 40-year-old male has recently been diagnosed with hypertension due to pressure readings of 163/102 and 165/100 mm Hg. He also has diabetes that is well controlled with oral hypoglycemic medications. Which is the best initial treatment regimen for treatment of hypertension in this patient? A. Felodipine. B. Furosemide. IT C. Lisinopril. D. Lisinopril and hydrochlorothiazide. E. Metoprolol. Correct answer = D. Because the systolic blood pressure is more than 20 mm Hg above goal (10 mm Hg above goal diastolic), treatment with two different medications is preferred. Because the patient is diabetic, he also has a compelling indication for an ACE inhibitor or ARB. 61- A 60-year-old white female has not reached her blood pressure goal after 1 month of treatment with a low dose of lisinopril. All of the following would be appropriate next steps in the treatment of her hypertension except: A. Increase dose of lisinopril. B. Add a diuretic medication. C. Add on a calcium channel blocker medication. D. Add on an ARB medication. Correct answer = D. Increasing the dose of lisinopril or adding a second medication from a different class (such as a calcium channel blocker or diuretic) would be appropriate steps to control the blood pressure. Adding an ARB as the second medication is not recommended. ARBs have a similar mechanism of action to ACE inhibitors, and combination therapy may increase the risk of adverse effects. 62- A patient returns to her health care provider for routine monitoring 3 months after her hypertension regimen was modified. Labs reveal elevated serum potassium. Which is likely responsible for this hyperkalemia? A. Chlorthalidone. B. Clonidine. FAR C. Furosemide. E ar D. Losartan. E. Nifedipine. Hiba Alzoubi pct 28 | P a g e Correct answer = D. Losartan, an ARB, can cause an increase in serum potassium similar to ACE inhibitors. Furosemide and chlorthalidone can cause a decrease in serum potassium. Nifedipine and clonidine do not affect potassium levels. 63- A 58-year-old female reports that she recently stopped taking her blood pressure medications because of swelling in her feet that began shortly after she started treatment. Which is most likely to cause peripheral edema? A. Atenolol. B. Clonidine. C. Felodipine. D. Hydralazine. CCB E. Prazosin. Correct answer = C. Peripheral edema is one of the most common side effects of calcium channel blockers. None of the other agents commonly cause peripheral edema. 64- DD is a 50-year-old male with newly diagnosed hypertension. His comorbidities include diabetes and chronic hepatitis C infection with moderate liver impairment. He requires two drugs for initial treatment of his hypertension. Which should be prescribed in combination with a thiazide diuretic? A. Lisinopril. B. Spironolactone. C. Fosinopril. ACE Prevent D. Furosemide. E. Hydralazine. nephropathy Correct answer = A. Because DD has diabetes, he has a compelling indication for an ACE inhibitor or ARB for the treatment of his hypertension and prevention of diabetic nephropathy. However, most ACE inhibitors undergo hepatic conversion e to active metabolites, so his hepatic impairment is of concern. Because lisinopril is It one of the two ACE inhibitors that does not undergo hepatic conversion to active metabolites, it is the best choice. Fosinopril is the only ACE inhibitor that is not eliminated primarily by the kidneys but does undergo hepatic conversion. An additional diuretic like spironolactone or furosemide is not indicated. DD does not have a compelling indication for hydralazine. Li si no Pri s hepatic Hiba Alzoubi j saw 29 | P a g e Chapter 18: Diuretics 65- An elderly patient with a history of heart disease is brought to the emergency I room with difficulty breathing. Examination reveals that she has pulmonary edema. Which treatment is indicated? A. Acetazolamide. B. Chlorthalidone. C. Furosemide. no op D. Hydrochlorothiazide. E. Spironolactone. long and o XI g nation e Correct answer = C. This is a potentially fatal situation. It is important to administer a diuretic that will reduce fluid accumulation in the lungs and, thus, improve oxygenation and heart function. The loop diuretics are most effective in removing large fluid volumes from the body and are the treatment of choice in this situation. In this situation, furosemide should be administered intravenously. The other choices are inappropriate. 50 66- A group of college students is planning a mountain climbing trip to the Andes. Which would be appropriate for them to take to prevent mountain sickness? A. A thiazide diuretic such as hydrochlorothiazide. B. An anticholinergic such as atropine. C. A carbonic anhydrase inhibitor such as acetazolamide. D. A loop diuretic such as furosemide. E. A β-blocker such as metoprolol. Dean Correct answer = C. Acetazolamide is used prophylactically for several days before an ascent above 10,000 feet. This treatment prevents the cerebral and pulmonary problems associated with the syndrome as well as other difficulties, such as nausea. 67- An alcoholic male has developed hepatic cirrhosis. To control the ascites and edema, which should be prescribed? A. Acetazolamide. B. Chlorthalidone. C. Furosemide. D. Hydrochlorothiazide. loop as Palma nary E. Spironolactone. T hepatica K Carbon Anhydrite 30 | P a g e Hiba Alzoubi Aaaa Correct answer = E. Spironolactone is very effective in the treatment of hepatic edema. These patients are frequently resistant to the diuretic action of loop diuretics, although a combination with spironolactone may be beneficial. The other agents are not indicated. 68- A 55-year-old male with kidney stones has been placed on a diuretic to decrease calcium excretion. However, after a few weeks, he develops an attack of gout. Which diuretic was he taking? A. Furosemide. B. Hydrochlorothiazide. 1 C. Spironolactone. a encreah PC D. Triamterene. Paton a E. Urea.1 2008 hxfectacinia Correct answer = B. Hydrochlorothiazide is effective in increasing calcium reabsorption, thus decreasing the amount of calcium excreted, and decreasing the formation of kidney stones that contain calcium phosphate or calcium oxalate. However, hydrochlorothiazide can also inhibit the excretion of uric acid and cause its accumulation, leading to an attack of gout in some individuals. Furosemide increases the excretion of calcium, whereas the K+-sparing osmotic diuretics, spironolactone and triamterene, and urea do not have an effect. 69- A 75-year-old woman with hypertension is being treated with a thiazide. Her blood pressure responds well and reads at 120/76 mm Hg. After several months on the medication, she complains of being tired and weak. An analysis of the blood FIS indicates low values for which of the following? a A. Calcium. B. Glucose. n mad C. Potassium. p É D. Sodium. E. Uric acid. abba I Correct answer = C. Hypokalemia is a common adverse effect of the thiazides and causes fatigue and lethargy in the patient. Supplementation with potassium chloride or foods high in K+ corrects the problem. Alternatively, a potassiumsparing diuretic, such as spironolactone, may be added. Calcium, uric acid, and glucose are usually elevated by thiazide diuretics. Sodium loss would not weaken the patient. are 31 | P a g e Hiba Alzoubi 70- A male patient is placed on a new medication and notes that his breasts have become enlarged and tender to the touch. Which medication is he most likely taking? A. Chlorthalidone. B. Furosemide. C. Hydrochlorothiazide. D. Spironolactone. E. Triamterene. Correct answer = D. An adverse drug reaction to spironolactone is gynecomastia due to its effects on androgens and progesterone in the body. Eplerenone may be a suitable alternative if the patient is in need of an aldosterone antagonist but has a history of gynecomastia. 71- A patient presents to the emergency department with an extreme headache. After a thorough workup, the attending physician concludes that the pain is due to increased intracranial pressure. Which diuretic would work best to reduce this pressure? A. Acetazolamide. B. Indapamide. C. Furosemide. 3 e D. Hydrochlorothiazide. BJ aIe E. Mannitol. Correct answer = E. Osmotic diuretics, such as mannitol, are a mainstay of treatment for patients with increased intracranial pressure or acute renal failure due to shock, drug toxicities, and trauma. og Chapter 19: Heart Failure 72- BC is a 70-year-old female who is diagnosed with HFrEF. Her past medical history is significant for hypertension and atrial fibrillation. She is taking hydrochlorothiazide, lisinopril, metoprolol tartrate, and warfarin. BC says she is feeling “good” and has no cough, shortness of breath, or edema. Which is the most appropriate medication change to make? A. Discontinue hydrochlorothiazide. B. Change lisinopril to losartan. C. Decrease warfarin dose. __ D D. Change metoprolol tartrate to metoprolol succinate. ___ 32 | P a g e Hiba Alzoubi O Correct answer = D. Metoprolol succinate should be used in HF, given that there is mortality benefit shown with metoprolol succinate in landmark HF trials. Hydrochlorothiazide and warfarin are appropriate based on the information given; there is no reason to change to an ARB since the patient has no cough or history of angioedema. y 73- SC is a 75-year-old white male who has HF. He is seen in clinic today, reporting shortness of breath, increased pitting edema, and a 5-pound weight gain over the last 2 days. His current medication regimen includes losartan and metoprolol succinate. SC has no chest pain and is deemed stable for outpatient treatment. Which of the following is the best recommendation? A. Increase the dose of metoprolol succinate. B. Start hydrochlorothiazide. C. Start furosemide. D. Discontinue losartan. Correct answer = C. As it is possible that SC is having a HF exacerbation, increasing the dose of the β-blocker is not indicated at this time. There is no reason to stop losartan, based on the information we have. Loop diuretics are preferred over thiazide diuretics when patients require diuresis immediately. Chapter 20: Antiarrhythmics 74- A 60-year-old woman had a myocardial infarction. Which of the following should be used to prevent life-threatening arrhythmias that can occur post– myocardial infarction in this patient? A. Digoxin. B. Flecainide. C. Metoprolol. D. Procainamide. E. Quinidine. Correct answer = C. β-Blockers such as metoprolol prevent arrhythmias that occur subsequent to a myocardial infarction. None of the other drugs has been shown to be effective in preventing postinfarct arrhythmias. Flecainide should be avoided in patients with structural heart disease. 33 | P a g e Hiba Alzoubi 75- A 57-year-old man is being treated for an atrial arrhythmia. He complains of dry mouth, blurred vision, and urinary hesitancy. Which antiarrhythmic drug is he mostly like taking? A. Metoprolol. B. Disopyramide. C. Dronedarone. D. Sotalol. Correct answer = B. The clustered symptoms of dry mouth, blurred vision, and urinary hesitancy are characteristic of anticholinergic adverse effects which are caused by class IA agents (in this case, disopyramide). The other drugs do not cause anticholinergic effects. 76- A 58-year-old woman is being treated for chronic suppression of a ventricular arrhythmia. After 1 week of therapy, she complains about feeling severe upset stomach and heartburn. Which antiarrhythmic drug is the likely cause of these symptoms? A. Amiodarone. B. Digoxin. C. Mexiletine. D. Propranolol. E. Quinidine. Correct answer = C. The patient is exhibiting a classic adverse effect of mexiletine. None of the other agents listed are likely to cause dyspepsia. 77- A 78-year-old woman has been newly diagnosed with atrial fibrillation. She is not currently having symptoms of palpitations or fatigue. Which is appropriate to initiate for rate control as an outpatient? A. Amiodarone. B. Dronedarone. C. Esmolol. D. Flecainide. E. Metoprolol. Correct answer = E. Only C and E are options to control rate. The other options are used for rhythm control in patients with atrial fibrillation. Since esmolol is IV only, the only option to start as an outpatient is metoprolol. 34 | P a g e Hiba Alzoubi 78- A clinician would like to initiate a drug for rhythm control of atrial fibrillation. Which of the following coexisting conditions would allow for initiation of flecainide? A. Hypertension. B. Left ventricular hypertrophy. C. Coronary artery disease. D. Heart failure. Correct answer = A. Since flecainide can increase the risk of sudden cardiac death in those with a history of structural heart disease, only A will allow for flecainide initiation. Structural heart disease includes left ventricular hypertrophy, heart failure, and atherosclerotic heart disease. Chapter 21: Antianginal Drugs 79- A 72-year-old male presents to the primary care clinic complaining of chest tightness and pressure that is increasing in severity and frequency. His current medications include atenolol, lisinopril, and nitroglycerin. Which intervention is most appropriate at this time? A. Add amlodipine. B. Initiate isosorbide mononitrate. C. Initiate ranolazine. D. Refer the patient to the nearest emergency room for evaluation. Correct answer = D. Crescendo angina is indicative of unstable angina that requires further workup. 80- A 62-year-old patient with a history of asthma and vasospastic angina states that he gets chest pain both with exertion and at rest, about ten times per week. One sublingual nitroglycerin tablet always relieves his symptoms, but this medication gives him an awful headache every time he takes it. Which is the best option for improving his angina? A. Change to sublingual nitroglycerin spray. B. Add amlodipine. C. Add propranolol. D. Replace nitroglycerin with ranolazine. Correct answer = B. Calcium channel blockers are preferred for vasospastic angina. β-Blockers can actually worsen vasospastic angina; furthermore, nonselective β-blockers should be avoided in patients with asthma. The nitroglycerin spray would also be expected to cause headache, so this is not the 35 | P a g e Hiba Alzoubi best choice. Ranolazine is not indicated for immediate relief of an angina attack, nor is it a first-line option. 81- A 65-year-old male experiences uncontrolled angina attacks that limit his ability to do household chores. He is adherent to a maximized dose of β-blocker with a low heart rate and low blood pressure. He was unable to tolerate an increase in isosorbide mononitrate due to headache. Which is the most appropriate addition to his antianginal therapy? A. Amlodipine. B. Aspirin. C. Ranolazine. D. Verapamil. Correct answer = C. Ranolazine is the best answer. The patient’s blood pressure is low, so verapamil and amlodipine may drop blood pressure further. Verapamil may also decrease heart rate. Ranolazine can be used when other agents are maximized, especially when blood pressure is well controlled. The patient will need a baseline ECG and lab work to ensure safe use of this medication. 82- A 68-year-old male with a history of angina had a MI last month, and an echocardiogram reveals heart failure with reduced ejection fraction. He was continued on his previous home medications (diltiazem, enalapril, and nitroglycerin), and atenolol was added at discharge. He has only had a few sporadic episodes of stable angina that are relieved with nitroglycerin or rest. What are eventual goals for optimizing this medication regimen? A. Add isosorbide mononitrate. B. Increase atenolol. C. Stop atenolol and increase diltiazem. D. Stop diltiazem and change atenolol to bisoprolol. Correct answer = D. Nondihydropyridine calcium channel blockers such as diltiazem should be avoided in patients with heart failure with reduced ejection fraction. Patients should be treated with one of three β-blockers approved for heart failure with reduced ejection fraction (bisoprolol, metoprolol succinate, or carvedilol). It sounds like his angina symptoms are well managed with his current therapy so adding isosorbide mononitrate would not be necessary. These symptoms may become even less frequent as his new β-blocker is titrated. 83- A patient whose angina was previously well controlled with once-daily isosorbide mononitrate states that recently he has been taking isosorbide 36 | P a g e Hiba Alzoubi mononitrate twice a day to control angina symptoms that are occurring more frequently during early morning hours. Which of the following is the best option for this patient? A. Continue once-daily administration of isosorbide mononitrate but advise the patient to take this medication in the evening. B. Advise continuation of isosorbide mononitrate twice daily for full 24-hour coverage of anginal symptoms. C. Switch to isosorbide dinitrate, as this has a longer duration of action than the mononitrate. D. Switch to nitroglycerin patch for consistent drug delivery and advise him to wear the patch around the clock. Correct answer = A. It is important to maintain a nitrate-free period to prevent the development of tolerance to nitrate therapy. The mononitrate formulation has the longer halflife. The nitroglycerin patch should be taken off for 10 to 12 hours daily to allow for nitrate-free interval. Chapter 22: Anticoagulants and Antiplatelet Agents 84- A 70-year-old female is diagnosed with nonvalvular atrial fibrillation. Her past medical history is significant for chronic kidney disease, and her renal function is moderately diminished. All of the following anticoagulants would be expected to require a reduced dosage in this patient except: A. Apixaban. B. Dabigatran. C. Rivaroxaban. D. Warfarin. Correct answer = D. Warfarin does not require dosage adjustment in renal dysfunction. The INR is monitored and dosage adjustments are made on the basis of this information. All of the other agents are renally cleared to some extent and require dosage adjustments in renal dysfunction. 85- An 80-year-old male is taking warfarin indefinitely for the prevention of deep venous thrombosis. He is a compliant patient with a stable INR and has no issues with bleeding or bruising. He is diagnosed with a urinary tract infection and is prescribed sulfamethoxazole/ trimethoprim. What effect will this have on his warfarin therapy? 37 | P a g e Hiba Alzoubi A. Sulfamethoxazole/trimethoprim will decrease the anticoagulant effect of warfarin. B. Sulfamethoxazole/trimethoprim will increase the anticoagulant effect of warfarin. C. Sulfamethoxazole/trimethoprim will activate platelet activity. D. Sulfamethoxazole/trimethoprim will not change anticoagulation status. Correct answer = B. Sulfamethoxazole/trimethoprim has a significant drug interaction with warfarin, such that it will inhibit warfarin metabolism. Therefore, sulfamethoxazole/ trimethoprim will cause increased anticoagulation, and the patient will need to have his warfarin dose decreased and INR checked frequently while he is on this antibiotic. 86- A 56-year-old man presents to the emergency room with complaints of swelling, redness, and pain in his right leg. The patient is diagnosed with acute DVT and requires treatment with an anticoagulant. All of the following are approved for treatment of this patient’s DVT except: A. Rivaroxaban. B. Dabigatran. C. Enoxaparin. D. Heparin. Correct answer = B. Dabigatran is only approved for the prevention of stroke in nonvalvular atrial fibrillation; it is not approved for the treatment of acute DVT. All of the other options are approved for treatment of acute DVT. 87- A 62-year-old male taking warfarin for stroke prevention in atrial fibrillation presents to his primary care physician with an elevated INR of 10.5 without bleeding. He is instructed to hold his warfarin dose and given 2.5 mg of oral vitamin K1. When would the effects of vitamin K on the INR most likely be noted in this patient? A. 1 hour. B. 6 hours. C. 24 hours. D. 72 hours. Correct answer = C. Vitamin K1 takes about 24 hours to see a reduction in the INR. This is due to the time required for the body to synthesize new coagulation factors. 38 | P a g e Hiba Alzoubi 88- A 58-year-old man receives intravenous alteplase treatment for acute stroke. Five minutes following completion of alteplase infusion, he develops orolingual angioedema. Which of the following drugs may have increased the risk of developing orolingual angioedema in this patient? A. ACE inhibitor. B. GP IIb/IIIa receptor antagonist. C. Phosphodiesterase inhibitor. D. Thiazide diuretic. Correct answer = A. ACE inhibitors, aspirin, and prasugrel all have possible adverse effects including orolingual angioedema. In the setting of alteplase administration, ACE inhibitors have been associated with an increased risk of developing orolingual angioedema with concomitant use. Chapter 23: Drugs for Hyperlipidemia 89- JS is a 65-year-old man who presents to his physician for management of hyperlipidemia. His most recent lipid panel reveals an LDL cholesterol level of 165 mg/ dL. His physician wishes to begin treatment to lower his LDL cholesterol levels. Which of the following therapies is the best option to lower JS’s LDL cholesterol levels? A. Fenofibrate. B. Colesevelam. C. Niacin. D. Simvastatin. E. Ezetimibe. Correct answer = D. Simvastatin, an HMG CoA reductase inhibitor (statin), is the most effective option for lowering LDL cholesterol, achieving reductions of 30% to 41% from baseline levels. Fenofibrate and niacin are more effective at lowering triglyceride levels or raising HDL levels (niacin). Colesevelam can reduce LDL levels but not as effectively as statins. Ezetimibe lowers LDL levels modestly compared to the LDL reduction achieved by statins. 90- WW is a 62-year-old female with hyperlipidemia and hypothyroidism. Her current medications include cholestyramine and levothyroxine (thyroid hormone). What advice would you give to WW to avoid a drug interaction between her cholestyramine and levothyroxine? A. Stop taking the levothyroxine as it can interact with cholestyramine. B. Take levothyroxine 1 hour before cholestyramine on an empty stomach. 39 | P a g e Hiba Alzoubi C. Switch cholestyramine to colesevelam as this will eliminate the interaction. D. Switch cholestyramine to colestipol as this will eliminate the interaction. E. Take levothyroxine and cholestyramine at the same time to minimize the interaction. Correct answer = B. Cholestyramine and the bile acid resins can bind several medications causing decreased absorption. Cholestyramine can decrease absorption of medications such as levothyroxine. Taking levothyroxine 1 hour before or 4 to 6 hours after cholestyramine can help to avoid this interaction. Choices C and D are incorrect, as all bile acid resins cause this interaction. Choice A is incorrect, as this patient should not stop her thyroid medication. Choice E will worsen this drug interaction. 91- AJ is a 42-year-old man who was started on niacin sustained-release tablets 2 weeks ago for elevated triglycerides and low HDL levels. He is complaining of an uncomfortable flushing and itchy feeling that he thinks is related to the niacin. Which of the following options can help AJ manage this adverse effect of niacin therapy? A. Administer aspirin 30 minutes prior to taking niacin. B. Administer aspirin 30 minutes after taking niacin. C. Increase the dose of niacin SR to 1000 mg. D. Continue the current dose of niacin. E. Change the sustained-release niacin to immediate-release niacin. Correct answer = A. Flushing associated with niacin is prostaglandin mediated; therefore, use of aspirin (a prostaglandin inhibitor) can help to minimize this adverse effect. It must be administered 30 minutes prior to the dose of the niacin; therefore, choice B is incorrect. Increasing the dose of niacin is likely to increase these complaints; therefore, choice C is incorrect. Continuing the current dose is unlikely to relieve these complaints, which are bothersome to AJ. The sustained-release formulation of niacin has less incidence of flushing versus that of the immediate release; therefore, choice E is incorrect. 92- CN is a 72-year-old male who is treated for hyperlipidemia with high-dose atorvastatin for the past 6 months. He also has a history of renal insufficiency. His most recent lipid panel shows an LDL cholesterol level of 131 mg/dL, triglycerides of 510 mg/dL, and HDL cholesterol of 32 mg/dL. His physician wishes to add an additional agent for his hyperlipidemia. Which of the following choices is the best option to address CN’s dyslipidemia? A. Fenofibrate. 40 | P a g e Hiba Alzoubi B. Niacin. C. Colesevelam. D. Gemfibrozil. E. Ezetimibe. Correct answer = B. This patient has significantly elevated triglycerides and low HDL. Niacin can lower triglycerides by 35% to 50% and also raise HDL levels. The fibrates (fenofibrate and gemfibrozil) should not be used due to CN’s history of renal insufficiency. Use of colesevelam is contraindicated because triglycerides are greater than 400 mg/dL. Ezetimibe can further lower LDL cholesterol but has modest effects on triglycerides versus niacin. UNIT V: Drugs Affecting the Endocrine System Chapter 24: Pituitary and Thyroid 93- A 40-year-old female is undergoing infertility treatments. Which of the following drugs might be included in her treatment regimen? A. Cabergoline. B. Follitropin. C. Methimazole. D. Vasopressin. Correct answer = B. Follitropin is a recombinant version of FSH that causes ovarian follicular growth and maturation. Cabergoline is a dopamine agonist that is used for hyperprolactinemia. Methimazole is the treatment of choice for hyperthyroidism. Vasopressin is an antidiuretic hormone. Chapter 25: Drugs for Diabetes 94- DW is a patient with type 2 diabetes who has a blood glucose of 400 mg/dL today at his office visit. The physician would like to give some insulin to bring the glucose down before he leaves the office. Which of the following would lower the glucose in the quickest manner in DW? A. Insulin aspart. B. Insulin glargine. C. NPH insulin. D. Regular insulin. Correct answer = A. Insulin aspart is a rapid-acting insulin that has an onset of action within 15 to 20 minutes. Insulin glargine is a long-acting insulin that is used for basal control. NPH insulin is an intermediate-acting insulin that is used for 41 | P a g e Hiba Alzoubi basal control. Although regular insulin can be used to bring the glucose down, its onset is not as quick as insulin aspart. The onset of regular insulin is about 30 to 60 minutes. 95- A 64-year-old woman with a history of type 2 diabetes is diagnosed with heart failure. Which of the following medications would be a poor choice for controlling her diabetes? A. Exenatide. B. Glyburide. C. Nateglinide. D. Pioglitazone. E. Sitagliptin. Correct answer = D. The TZDs (pioglitazone and rosiglitazone) can cause fluid retention and lead to a worsening of heart failure. They should be used with caution and dose reduction, if at all, in patients with heart failure. Exenatide, glyburide, nateglinide, and sitagliptin do not have precautions for use in heart failure patients. 96- KD is a 69-year-old male with type 2 diabetes and advanced chronic kidney disease. Which of the following diabetes medications is contraindicated in this patient? A. Glipizide. B. Insulin lispro. C. Metformin. D. Saxagliptin. Correct answer = C. Metformin should not be used in patients with kidney disease due to the possibility of lactic acidosis. Glipizide can be used safely in patients with CrCl as low as 10 mL/min. Insulin is not contraindicated in renal dysfunction, although the dosage may need to be adjusted. While the dose of the DPP-4 inhibitor saxagliptin may need to be reduced in renal dysfunction, it is not contraindicated. 97- A patient with type 2 diabetes is taking metformin. The fasting glucose levels are in range, but the postprandial glucose is uncontrolled. All of the following drugs would be appropriate to add to metformin to target postprandial glucose except: A. Acarbose. B. Exenatide. 42 | P a g e Hiba Alzoubi C. Insulin aspart. D. Pramlintide. The correct answer = D. Although all of these drugs target postprandial glucose, pramlintide should only be used in conjunction with mealtime insulin. Since this patient is not on insulin, pramlintide is not indicated. Chapter 26: Estrogens and Androgens 98- A 53-year-old woman has severe vasomotor symptoms (hot flushes) associated with menopause. She has no pertinent past medical or surgical history. Which of the following would be most appropriate for her symptoms? A. Conjugated estrogens vaginal cream. B. Estradiol transdermal patch. C. Oral estradiol and medroxyprogesterone acetate. D. Injectable medroxyprogesterone acetate. Correct answer = C. Estrogen vaginal cream only treats vaginal symptoms of menopause such as vaginal atrophy and does not treat hot flushes. Since this patient has an intact uterus, a progestin such as medroxyprogesterone needs to be used along with the estrogen to prevent the development of endometrial hyperplasia. Unopposed estrogen (for example, the estradiol transdermal patch) should not be used. Injectable medroxyprogesterone acetate is used for contraception. 99- A 70-year-old woman is being treated with raloxifene for osteoporosis. Which of the following is a concern with this therapy? A. Breast cancer. B. Endometrial cancer. C. Venous thrombosis. D. Hypercholesterolemia. Correct answer = C. Raloxifene can increase the risk of venous thromboembolism. Unlike estrogen and tamoxifen, raloxifene does not result in an increased incidence of endometrial cancer. Raloxifene lowers the risk of breast cancer in high-risk women, and it also lowers LDL cholesterol. 100- A 26-year-old female is using injectable medroxyprogesterone acetate as a method of contraception. Which of the following adverse effects is a concern if she wishes to use this therapy long-term? 43 | P a g e Hiba Alzoubi A. Hyperkalemia. B. Male pattern baldness. C. Osteoporosis. D. Weight loss. Correct answer = C. Medroxyprogesterone acetate may contribute to bone loss and predispose patients to osteoporosis and/or fractures. Therefore, the drug should not be continued for more than 2 years if possible. The drug often causes weight gain, not weight loss. The other adverse effects are not associated with medroxyprogesterone. Chapter 27: Adrenal Hormones 101- A child with severe asthma is being treated with high doses of inhaled corticosteroids. Which of the following adverse effects is of particular concern? A. Hypoglycemia. B. Hirsutism. C. Growth suppression. D. Cushing syndrome. E. Cataract formation. Correct answer = C. Corticosteroids may retard bone growth. Chronic treatment with the medication therefore may lead to growth suppression, so linear growth should be monitored periodically. Hyperglycemia, not hypoglycemia, is a possible adverse effect. Hirsutism, Cushing syndrome, and cataract formation are unlikely with the dose that the child would receive by inhalation. 102- A patient with Addison disease is being treated with hydrocortisone but is still having problems with dehydration and hyponatremia. Which of the following drugs would be best to add to the patient’s therapy? A. Dexamethasone. B. Fludrocortisone. C. Prednisone. D. Triamcinolone. Correct answer = B. To combat dehydration and hyponatremia, a corticosteroid with high mineralocorticoid activity is needed. Fludrocortisone has the greatest mineralocorticoid activity of the agents provided. The other drugs have little or no mineralocorticoid activity. 44 | P a g e Hiba Alzoubi Chapter 28: Drugs for Obesity 103- A 45-year-old female presents seeking treatment for weight loss. She has tried several fad diets in the past with very little success. She exercises twice weekly at the gym for 30 minutes and tries to watch what she eats. Her BMI is 31 and she has diabetes and uncontrolled hypertension. Which of the following medications would be most appropriate to treat her obesity? A. Phentermine. B. Phentermine/topiramate. C. Orlistat. D. Diethylpropion. Correct answer = C. Orlistat is the only medication of those listed that does not increase heart rate and blood pressure. Since this patient’s blood pressure is currently uncontrolled, choosing a drug that does not affect blood pressure would be best at this time. 104- A 38-year-old obese male with depression is considering a weight loss medication following several failed attempts with diet and exercise. Which of the following medications should be avoided in this individual? A. Phentermine. B. Phentermine/topiramate. C. Orlistat. D. Diethylpropion. E. Lorcaserin. Correct answer = E. Lorcaserin may cause suicidal ideation and would not be advisable for an individual with depression. Also, he is likely on a medication that may increase serotonin levels. The addition of lorcaserin, a serotonin receptor agonist, could lead to serotonin syndrome. Therefore, avoidance of the combination is advisable. 105- A 27-year-old recently married female is asking about treatment options for her obesity. She recently stopped taking her birth control medications, as she felt these were contributing to her weight gain. Which of the following medications should be avoided in this patient? A. Phentermine. B. Phentermine/topiramate. C. Orlistat. D. Diethylpropion. E. Lorcaserin. 45 | P a g e Hiba Alzoubi Correct answer = B. The topiramate component of this medication is contraindicated in pregnancy. Since this patient stopped her birth control, she is at risk of becoming pregnant and her fetus is at risk o